Inscription / Connexion Nouveau Sujet
Niveau autre
Partager :

Recherche mathématique

Posté par sambgoree (invité) 23-05-06 à 14:31

Bonjour
La relation a^x-b^y=1 admet-elle d'autres possibilités que 3^2-2^3=1

Posté par
J-P Posteur d'énigmes
re : Recherche mathématique 23-05-06 à 16:04

Il faudrait préciser à quels ensembles appartiennent a, b, x et y, sinon:

2189^0 - 0^{568974} = 1





Posté par
Nicolas_75 Correcteur
re : Recherche mathématique 23-05-06 à 17:05

Ou encore :
4^1-3^1=1

Posté par
J-P Posteur d'énigmes
re : Recherche mathématique 23-05-06 à 18:25

Je subodore que c'est a^b - b^a = 1 avec a et b dans Z, mais il faudrait que sambgoree confirme ou infirme.


Posté par sambgoree (invité)re : Recherche mathématique 23-05-06 à 21:16

Excusez-moi!, c'est parfait J-P,a^b-b^a=1,a et b \in \mathbb{Z}

Posté par
J-P Posteur d'énigmes
re : Recherche mathématique 24-05-06 à 14:58

1^0 - 0^1 = 1

2^0 - 0^2 = 1

3^0 - 0^3 = 1

...

Posté par sambgoree (invité)re : Recherche mathématique 24-05-06 à 15:08

Parfait!Je me demandais aussi, sia et b \in \mathbb{N^*}...est-ce une relation possible? En tout cas j'esseyerais de trouver une démonstration pour confirmer ou infirmer!...merci aussi à ceux qui m'aiderons!

Posté par
Cauchy
re : Recherche mathématique 24-05-06 à 16:32

Bonjour a tous ,

l'equation a^b-b^a=1\,\,\,(E) admet dans (\mathbb{N}^{*})^{2} pour seules solutions les couples (3,2) et (2,1).

Si on suppose a \geq 4 on va montrer qu'il n'y a pas de solution.

On a deja que a^b >b^a.De cela on tire :

\Large{a^{\frac{1}{a}} > b^{\frac{1}{b}}}.

En etudiant la fonction  \Large{x->x^{\frac{1}{x}}} on voit qu'elle est decroissante pour x>3 donc: \Large{2^{\frac{1}{2}}=4^{\frac{1}{4}} > 5^{\frac{1}{5}} > 6^{\frac{1}{6}}\cdots}.

De ces deux derniers points on en deduit que pour a \geq 4 ,on a : b \geq a+1. (Sinon on peut aussi avoir b=1 mais la ce n'est pas possible car a>3 et (E) ne peut etre verifie).

On pose c=b-a \geq 1.

On a donc :

\Large{\frac{a^b}{b^a}=\frac{a^{a+c}}{(a+c)^{a}}=\frac{a^c}{(1+\frac{c}{a})^{a}}}.

On utilise l'inegalité :\Large{e^t >1+t,t>0} avec \Large{t=\frac{c}{a}} d'ou : \Large{e^{\frac{c}{a}} > 1+ \frac{c}{a}} puis \Large{e^c > (1+ \frac{c}{a})^{a}}.

De la on tire :\Large{\frac{a^b}{b^a} > \frac{a^c}{e^c}={\left(\frac{a}{e}\right)}^{c} \geq \frac{a}{e} \geq \frac{4}{e} >1,4}.

Finalement : \Large{a^b-b^a > b^a(0.4)=4*\frac{b^a}{10} \geq 4*\frac{5^4}{10} >1}

Donc il n'y a pas de solution pour a \geq 4.
IL faut juste traiter les cas a=1,2,3 et on aboutit aux solutions (2,1) et (3,2).

Posté par sambgoree (invité)re : Recherche mathématique 25-05-06 à 12:31

Merci Cauchy, Quelle démonstration!!!



Vous devez être membre accéder à ce service...

Pas encore inscrit ?

1 compte par personne, multi-compte interdit !

Ou identifiez-vous :


Rester sur la page

Inscription gratuite

Fiches en rapport

parmi 1675 fiches de maths

Désolé, votre version d'Internet Explorer est plus que périmée ! Merci de le mettre à jour ou de télécharger Firefox ou Google Chrome pour utiliser le site. Votre ordinateur vous remerciera !